Finding Bounds for Area Between Polar Curves Announcing the arrival of Valued Associate #679: Cesar Manara Planned maintenance scheduled April 17/18, 2019 at 00:00UTC (8:00pm US/Eastern)Area Bounded by Polar CurvesFinding area between two polar curves using double integralsarea between two polar curvesCan someone check my answer for this area between 2 polar curves question?Area between two polar curves $r = 2 sintheta$ and $r =2costheta$Area of two polar regionsArea inside polar curveDetermining bounds for polar areaArea between polar curvesDetermining the area between two polar curves using a double integral

How to assign captions for two tables in LaTeX?

Withdrew £2800, but only £2000 shows as withdrawn on online banking; what are my obligations?

Should gear shift center itself while in neutral?

What makes black pepper strong or mild?

Why does Python start at index 1 when iterating an array backwards?

What are the motives behind Cersei's orders given to Bronn?

Should I use Javascript Classes or Apex Classes in Lightning Web Components?

Letter Boxed validator

What causes the vertical darker bands in my photo?

Bonus calculation: Am I making a mountain out of a molehill?

3 doors, three guards, one stone

What does the "x" in "x86" represent?

Is 1 ppb equal to 1 μg/kg?

I am not a queen, who am I?

When -s is used with third person singular. What's its use in this context?

How do I determine if the rules for a long jump or high jump are applicable for Monks?

Is there a Spanish version of "dot your i's and cross your t's" that includes the letter 'ñ'?

Why did the IBM 650 use bi-quinary?

Is it true that "carbohydrates are of no use for the basal metabolic need"?

Should I call the interviewer directly, if HR aren't responding?

List *all* the tuples!

What LEGO pieces have "real-world" functionality?

If Jon Snow became King of the Seven Kingdoms what would his regnal number be?

Antler Helmet: Can it work?



Finding Bounds for Area Between Polar Curves



Announcing the arrival of Valued Associate #679: Cesar Manara
Planned maintenance scheduled April 17/18, 2019 at 00:00UTC (8:00pm US/Eastern)Area Bounded by Polar CurvesFinding area between two polar curves using double integralsarea between two polar curvesCan someone check my answer for this area between 2 polar curves question?Area between two polar curves $r = 2 sintheta$ and $r =2costheta$Area of two polar regionsArea inside polar curveDetermining bounds for polar areaArea between polar curvesDetermining the area between two polar curves using a double integral










0












$begingroup$


I'm trying to find the area of the region both inside the circle $r= sinθ$ and outside the circle $r=√3 cosθ$ (both equations are in polar coordinates). Here is what it looks like: graph



The two graphs intersect at the origin and the polar point $(r, theta) = (pi/3, fracsqrt32)$.



I thought the obvious answer would be to use the formula $A = frac12int_theta_1^theta_2 [R^2 - r^2] dtheta$ and integrate from $0$ to $pi/3$ as seen below:



$$A = frac12int_0^pi/3 [(sintheta)^2 - (sqrt3costheta)^2] dtheta$$



however, this is wrong. What they have instead is:



$$A = frac12int_pi/3^pi (sintheta)^2 dtheta quad - quad frac12int_pi/3^pi/2(sqrt3costheta)^2] dtheta$$



Can someone please explain how they got the bounds in their answer? And if there is a simpler way, for example, a slight alteration to the way I initially tried to do it, please let me know.



Thank you!










share|cite|improve this question









$endgroup$











  • $begingroup$
    The first integral finds the area of the smaller circle lying between the rays $theta=pi/3$ and $theta=pi$. But that is too much by the sector of the large circle between the rays $theta=pi/3$ and $theta=pi/2$, so that part is subtracted away by the second integral.
    $endgroup$
    – John Wayland Bales
    Mar 31 at 23:37











  • $begingroup$
    You're not telling me where the $pi/2$ is coming from...
    $endgroup$
    – CodingMee
    Mar 31 at 23:58















0












$begingroup$


I'm trying to find the area of the region both inside the circle $r= sinθ$ and outside the circle $r=√3 cosθ$ (both equations are in polar coordinates). Here is what it looks like: graph



The two graphs intersect at the origin and the polar point $(r, theta) = (pi/3, fracsqrt32)$.



I thought the obvious answer would be to use the formula $A = frac12int_theta_1^theta_2 [R^2 - r^2] dtheta$ and integrate from $0$ to $pi/3$ as seen below:



$$A = frac12int_0^pi/3 [(sintheta)^2 - (sqrt3costheta)^2] dtheta$$



however, this is wrong. What they have instead is:



$$A = frac12int_pi/3^pi (sintheta)^2 dtheta quad - quad frac12int_pi/3^pi/2(sqrt3costheta)^2] dtheta$$



Can someone please explain how they got the bounds in their answer? And if there is a simpler way, for example, a slight alteration to the way I initially tried to do it, please let me know.



Thank you!










share|cite|improve this question









$endgroup$











  • $begingroup$
    The first integral finds the area of the smaller circle lying between the rays $theta=pi/3$ and $theta=pi$. But that is too much by the sector of the large circle between the rays $theta=pi/3$ and $theta=pi/2$, so that part is subtracted away by the second integral.
    $endgroup$
    – John Wayland Bales
    Mar 31 at 23:37











  • $begingroup$
    You're not telling me where the $pi/2$ is coming from...
    $endgroup$
    – CodingMee
    Mar 31 at 23:58













0












0








0





$begingroup$


I'm trying to find the area of the region both inside the circle $r= sinθ$ and outside the circle $r=√3 cosθ$ (both equations are in polar coordinates). Here is what it looks like: graph



The two graphs intersect at the origin and the polar point $(r, theta) = (pi/3, fracsqrt32)$.



I thought the obvious answer would be to use the formula $A = frac12int_theta_1^theta_2 [R^2 - r^2] dtheta$ and integrate from $0$ to $pi/3$ as seen below:



$$A = frac12int_0^pi/3 [(sintheta)^2 - (sqrt3costheta)^2] dtheta$$



however, this is wrong. What they have instead is:



$$A = frac12int_pi/3^pi (sintheta)^2 dtheta quad - quad frac12int_pi/3^pi/2(sqrt3costheta)^2] dtheta$$



Can someone please explain how they got the bounds in their answer? And if there is a simpler way, for example, a slight alteration to the way I initially tried to do it, please let me know.



Thank you!










share|cite|improve this question









$endgroup$




I'm trying to find the area of the region both inside the circle $r= sinθ$ and outside the circle $r=√3 cosθ$ (both equations are in polar coordinates). Here is what it looks like: graph



The two graphs intersect at the origin and the polar point $(r, theta) = (pi/3, fracsqrt32)$.



I thought the obvious answer would be to use the formula $A = frac12int_theta_1^theta_2 [R^2 - r^2] dtheta$ and integrate from $0$ to $pi/3$ as seen below:



$$A = frac12int_0^pi/3 [(sintheta)^2 - (sqrt3costheta)^2] dtheta$$



however, this is wrong. What they have instead is:



$$A = frac12int_pi/3^pi (sintheta)^2 dtheta quad - quad frac12int_pi/3^pi/2(sqrt3costheta)^2] dtheta$$



Can someone please explain how they got the bounds in their answer? And if there is a simpler way, for example, a slight alteration to the way I initially tried to do it, please let me know.



Thank you!







calculus area polar-coordinates






share|cite|improve this question













share|cite|improve this question











share|cite|improve this question




share|cite|improve this question










asked Mar 31 at 23:26









CodingMeeCodingMee

756




756











  • $begingroup$
    The first integral finds the area of the smaller circle lying between the rays $theta=pi/3$ and $theta=pi$. But that is too much by the sector of the large circle between the rays $theta=pi/3$ and $theta=pi/2$, so that part is subtracted away by the second integral.
    $endgroup$
    – John Wayland Bales
    Mar 31 at 23:37











  • $begingroup$
    You're not telling me where the $pi/2$ is coming from...
    $endgroup$
    – CodingMee
    Mar 31 at 23:58
















  • $begingroup$
    The first integral finds the area of the smaller circle lying between the rays $theta=pi/3$ and $theta=pi$. But that is too much by the sector of the large circle between the rays $theta=pi/3$ and $theta=pi/2$, so that part is subtracted away by the second integral.
    $endgroup$
    – John Wayland Bales
    Mar 31 at 23:37











  • $begingroup$
    You're not telling me where the $pi/2$ is coming from...
    $endgroup$
    – CodingMee
    Mar 31 at 23:58















$begingroup$
The first integral finds the area of the smaller circle lying between the rays $theta=pi/3$ and $theta=pi$. But that is too much by the sector of the large circle between the rays $theta=pi/3$ and $theta=pi/2$, so that part is subtracted away by the second integral.
$endgroup$
– John Wayland Bales
Mar 31 at 23:37





$begingroup$
The first integral finds the area of the smaller circle lying between the rays $theta=pi/3$ and $theta=pi$. But that is too much by the sector of the large circle between the rays $theta=pi/3$ and $theta=pi/2$, so that part is subtracted away by the second integral.
$endgroup$
– John Wayland Bales
Mar 31 at 23:37













$begingroup$
You're not telling me where the $pi/2$ is coming from...
$endgroup$
– CodingMee
Mar 31 at 23:58




$begingroup$
You're not telling me where the $pi/2$ is coming from...
$endgroup$
– CodingMee
Mar 31 at 23:58










2 Answers
2






active

oldest

votes


















2












$begingroup$

You seem to have a misunderstanding about how the curves in the figure are drawn, and what part of the enclosed area comprises the region of interest.



Refer to the following animation, which sweeps out the curves for $0 le theta le pi$.



enter image description here



You can see that when $0 le theta < pi/3$, the portion of the curves drawn are the outer arc of the orange circle (where "outer" means that this arc is outside the blue circle) and the inner arc of the blue circle (where "inner" means this arc is inside the orange circle).



Only when $pi/3 le theta le pi$ do the curves swept by the black line represent the region you wish to integrate: specifically, as you stated, inside the circle $r = sin theta$ and outside the circle $r = sqrt3 cos theta$. However, an important subtlety occurs when $theta = pi/2$: This is where $r < 0$ for the orange circle. This is why the integral is split up at this point, since if you apply your formula blindly, you would find yourself unnecessarily subtracting the orange circle's contribution from the blue circle from $pi/2 < theta < pi$.



To recap, the area inside the blue circle and outside the orange comprises the wedge-shaped region in the first quadrant $$frac12int_theta = pi/3^pi/2 ((sin theta)^2 - (sqrt3 cos theta)^2) , dtheta,$$ since the blue curve's distance from the origin is greater than the orange curve's for angles in that interval; and the semicircular region $$frac12 int_theta = pi/2^pi (sin theta)^2 , dtheta.$$ What the provided solution has simply done is taken the blue curve's total contribution and subtracted the orange--it is simply an algebraic rearrangement.






share|cite|improve this answer









$endgroup$












  • $begingroup$
    I had to keep coming back and showing this answer to a lot of people, but I finally understand. Thank you!
    $endgroup$
    – CodingMee
    Apr 4 at 20:20


















0












$begingroup$

The intersection for $x$ correspond to
$$sin(theta)cos(theta)=sqrt 3 cos(theta)cos(theta)$$
$$cos(theta)big(sin(theta)-sqrt 3 cos(theta)big)=0$$ then the two solutions corresponding to $cos(theta)=0$ and $sin(theta)-sqrt 3 cos(theta)=0$






share|cite|improve this answer









$endgroup$













    Your Answer








    StackExchange.ready(function()
    var channelOptions =
    tags: "".split(" "),
    id: "69"
    ;
    initTagRenderer("".split(" "), "".split(" "), channelOptions);

    StackExchange.using("externalEditor", function()
    // Have to fire editor after snippets, if snippets enabled
    if (StackExchange.settings.snippets.snippetsEnabled)
    StackExchange.using("snippets", function()
    createEditor();
    );

    else
    createEditor();

    );

    function createEditor()
    StackExchange.prepareEditor(
    heartbeatType: 'answer',
    autoActivateHeartbeat: false,
    convertImagesToLinks: true,
    noModals: true,
    showLowRepImageUploadWarning: true,
    reputationToPostImages: 10,
    bindNavPrevention: true,
    postfix: "",
    imageUploader:
    brandingHtml: "Powered by u003ca class="icon-imgur-white" href="https://imgur.com/"u003eu003c/au003e",
    contentPolicyHtml: "User contributions licensed under u003ca href="https://creativecommons.org/licenses/by-sa/3.0/"u003ecc by-sa 3.0 with attribution requiredu003c/au003e u003ca href="https://stackoverflow.com/legal/content-policy"u003e(content policy)u003c/au003e",
    allowUrls: true
    ,
    noCode: true, onDemand: true,
    discardSelector: ".discard-answer"
    ,immediatelyShowMarkdownHelp:true
    );



    );













    draft saved

    draft discarded


















    StackExchange.ready(
    function ()
    StackExchange.openid.initPostLogin('.new-post-login', 'https%3a%2f%2fmath.stackexchange.com%2fquestions%2f3170015%2ffinding-bounds-for-area-between-polar-curves%23new-answer', 'question_page');

    );

    Post as a guest















    Required, but never shown

























    2 Answers
    2






    active

    oldest

    votes








    2 Answers
    2






    active

    oldest

    votes









    active

    oldest

    votes






    active

    oldest

    votes









    2












    $begingroup$

    You seem to have a misunderstanding about how the curves in the figure are drawn, and what part of the enclosed area comprises the region of interest.



    Refer to the following animation, which sweeps out the curves for $0 le theta le pi$.



    enter image description here



    You can see that when $0 le theta < pi/3$, the portion of the curves drawn are the outer arc of the orange circle (where "outer" means that this arc is outside the blue circle) and the inner arc of the blue circle (where "inner" means this arc is inside the orange circle).



    Only when $pi/3 le theta le pi$ do the curves swept by the black line represent the region you wish to integrate: specifically, as you stated, inside the circle $r = sin theta$ and outside the circle $r = sqrt3 cos theta$. However, an important subtlety occurs when $theta = pi/2$: This is where $r < 0$ for the orange circle. This is why the integral is split up at this point, since if you apply your formula blindly, you would find yourself unnecessarily subtracting the orange circle's contribution from the blue circle from $pi/2 < theta < pi$.



    To recap, the area inside the blue circle and outside the orange comprises the wedge-shaped region in the first quadrant $$frac12int_theta = pi/3^pi/2 ((sin theta)^2 - (sqrt3 cos theta)^2) , dtheta,$$ since the blue curve's distance from the origin is greater than the orange curve's for angles in that interval; and the semicircular region $$frac12 int_theta = pi/2^pi (sin theta)^2 , dtheta.$$ What the provided solution has simply done is taken the blue curve's total contribution and subtracted the orange--it is simply an algebraic rearrangement.






    share|cite|improve this answer









    $endgroup$












    • $begingroup$
      I had to keep coming back and showing this answer to a lot of people, but I finally understand. Thank you!
      $endgroup$
      – CodingMee
      Apr 4 at 20:20















    2












    $begingroup$

    You seem to have a misunderstanding about how the curves in the figure are drawn, and what part of the enclosed area comprises the region of interest.



    Refer to the following animation, which sweeps out the curves for $0 le theta le pi$.



    enter image description here



    You can see that when $0 le theta < pi/3$, the portion of the curves drawn are the outer arc of the orange circle (where "outer" means that this arc is outside the blue circle) and the inner arc of the blue circle (where "inner" means this arc is inside the orange circle).



    Only when $pi/3 le theta le pi$ do the curves swept by the black line represent the region you wish to integrate: specifically, as you stated, inside the circle $r = sin theta$ and outside the circle $r = sqrt3 cos theta$. However, an important subtlety occurs when $theta = pi/2$: This is where $r < 0$ for the orange circle. This is why the integral is split up at this point, since if you apply your formula blindly, you would find yourself unnecessarily subtracting the orange circle's contribution from the blue circle from $pi/2 < theta < pi$.



    To recap, the area inside the blue circle and outside the orange comprises the wedge-shaped region in the first quadrant $$frac12int_theta = pi/3^pi/2 ((sin theta)^2 - (sqrt3 cos theta)^2) , dtheta,$$ since the blue curve's distance from the origin is greater than the orange curve's for angles in that interval; and the semicircular region $$frac12 int_theta = pi/2^pi (sin theta)^2 , dtheta.$$ What the provided solution has simply done is taken the blue curve's total contribution and subtracted the orange--it is simply an algebraic rearrangement.






    share|cite|improve this answer









    $endgroup$












    • $begingroup$
      I had to keep coming back and showing this answer to a lot of people, but I finally understand. Thank you!
      $endgroup$
      – CodingMee
      Apr 4 at 20:20













    2












    2








    2





    $begingroup$

    You seem to have a misunderstanding about how the curves in the figure are drawn, and what part of the enclosed area comprises the region of interest.



    Refer to the following animation, which sweeps out the curves for $0 le theta le pi$.



    enter image description here



    You can see that when $0 le theta < pi/3$, the portion of the curves drawn are the outer arc of the orange circle (where "outer" means that this arc is outside the blue circle) and the inner arc of the blue circle (where "inner" means this arc is inside the orange circle).



    Only when $pi/3 le theta le pi$ do the curves swept by the black line represent the region you wish to integrate: specifically, as you stated, inside the circle $r = sin theta$ and outside the circle $r = sqrt3 cos theta$. However, an important subtlety occurs when $theta = pi/2$: This is where $r < 0$ for the orange circle. This is why the integral is split up at this point, since if you apply your formula blindly, you would find yourself unnecessarily subtracting the orange circle's contribution from the blue circle from $pi/2 < theta < pi$.



    To recap, the area inside the blue circle and outside the orange comprises the wedge-shaped region in the first quadrant $$frac12int_theta = pi/3^pi/2 ((sin theta)^2 - (sqrt3 cos theta)^2) , dtheta,$$ since the blue curve's distance from the origin is greater than the orange curve's for angles in that interval; and the semicircular region $$frac12 int_theta = pi/2^pi (sin theta)^2 , dtheta.$$ What the provided solution has simply done is taken the blue curve's total contribution and subtracted the orange--it is simply an algebraic rearrangement.






    share|cite|improve this answer









    $endgroup$



    You seem to have a misunderstanding about how the curves in the figure are drawn, and what part of the enclosed area comprises the region of interest.



    Refer to the following animation, which sweeps out the curves for $0 le theta le pi$.



    enter image description here



    You can see that when $0 le theta < pi/3$, the portion of the curves drawn are the outer arc of the orange circle (where "outer" means that this arc is outside the blue circle) and the inner arc of the blue circle (where "inner" means this arc is inside the orange circle).



    Only when $pi/3 le theta le pi$ do the curves swept by the black line represent the region you wish to integrate: specifically, as you stated, inside the circle $r = sin theta$ and outside the circle $r = sqrt3 cos theta$. However, an important subtlety occurs when $theta = pi/2$: This is where $r < 0$ for the orange circle. This is why the integral is split up at this point, since if you apply your formula blindly, you would find yourself unnecessarily subtracting the orange circle's contribution from the blue circle from $pi/2 < theta < pi$.



    To recap, the area inside the blue circle and outside the orange comprises the wedge-shaped region in the first quadrant $$frac12int_theta = pi/3^pi/2 ((sin theta)^2 - (sqrt3 cos theta)^2) , dtheta,$$ since the blue curve's distance from the origin is greater than the orange curve's for angles in that interval; and the semicircular region $$frac12 int_theta = pi/2^pi (sin theta)^2 , dtheta.$$ What the provided solution has simply done is taken the blue curve's total contribution and subtracted the orange--it is simply an algebraic rearrangement.







    share|cite|improve this answer












    share|cite|improve this answer



    share|cite|improve this answer










    answered Apr 1 at 4:14









    heropupheropup

    65.6k865104




    65.6k865104











    • $begingroup$
      I had to keep coming back and showing this answer to a lot of people, but I finally understand. Thank you!
      $endgroup$
      – CodingMee
      Apr 4 at 20:20
















    • $begingroup$
      I had to keep coming back and showing this answer to a lot of people, but I finally understand. Thank you!
      $endgroup$
      – CodingMee
      Apr 4 at 20:20















    $begingroup$
    I had to keep coming back and showing this answer to a lot of people, but I finally understand. Thank you!
    $endgroup$
    – CodingMee
    Apr 4 at 20:20




    $begingroup$
    I had to keep coming back and showing this answer to a lot of people, but I finally understand. Thank you!
    $endgroup$
    – CodingMee
    Apr 4 at 20:20











    0












    $begingroup$

    The intersection for $x$ correspond to
    $$sin(theta)cos(theta)=sqrt 3 cos(theta)cos(theta)$$
    $$cos(theta)big(sin(theta)-sqrt 3 cos(theta)big)=0$$ then the two solutions corresponding to $cos(theta)=0$ and $sin(theta)-sqrt 3 cos(theta)=0$






    share|cite|improve this answer









    $endgroup$

















      0












      $begingroup$

      The intersection for $x$ correspond to
      $$sin(theta)cos(theta)=sqrt 3 cos(theta)cos(theta)$$
      $$cos(theta)big(sin(theta)-sqrt 3 cos(theta)big)=0$$ then the two solutions corresponding to $cos(theta)=0$ and $sin(theta)-sqrt 3 cos(theta)=0$






      share|cite|improve this answer









      $endgroup$















        0












        0








        0





        $begingroup$

        The intersection for $x$ correspond to
        $$sin(theta)cos(theta)=sqrt 3 cos(theta)cos(theta)$$
        $$cos(theta)big(sin(theta)-sqrt 3 cos(theta)big)=0$$ then the two solutions corresponding to $cos(theta)=0$ and $sin(theta)-sqrt 3 cos(theta)=0$






        share|cite|improve this answer









        $endgroup$



        The intersection for $x$ correspond to
        $$sin(theta)cos(theta)=sqrt 3 cos(theta)cos(theta)$$
        $$cos(theta)big(sin(theta)-sqrt 3 cos(theta)big)=0$$ then the two solutions corresponding to $cos(theta)=0$ and $sin(theta)-sqrt 3 cos(theta)=0$







        share|cite|improve this answer












        share|cite|improve this answer



        share|cite|improve this answer










        answered Apr 1 at 4:25









        Claude LeiboviciClaude Leibovici

        126k1158135




        126k1158135



























            draft saved

            draft discarded
















































            Thanks for contributing an answer to Mathematics Stack Exchange!


            • Please be sure to answer the question. Provide details and share your research!

            But avoid


            • Asking for help, clarification, or responding to other answers.

            • Making statements based on opinion; back them up with references or personal experience.

            Use MathJax to format equations. MathJax reference.


            To learn more, see our tips on writing great answers.




            draft saved


            draft discarded














            StackExchange.ready(
            function ()
            StackExchange.openid.initPostLogin('.new-post-login', 'https%3a%2f%2fmath.stackexchange.com%2fquestions%2f3170015%2ffinding-bounds-for-area-between-polar-curves%23new-answer', 'question_page');

            );

            Post as a guest















            Required, but never shown





















































            Required, but never shown














            Required, but never shown












            Required, but never shown







            Required, but never shown

































            Required, but never shown














            Required, but never shown












            Required, but never shown







            Required, but never shown







            Popular posts from this blog

            Triangular numbers and gcdProving sum of a set is $0 pmod n$ if $n$ is odd, or $fracn2 pmod n$ if $n$ is even?Is greatest common divisor of two numbers really their smallest linear combination?GCD, LCM RelationshipProve a set of nonnegative integers with greatest common divisor 1 and closed under addition has all but finite many nonnegative integers.all pairs of a and b in an equation containing gcdTriangular Numbers Modulo $k$ - Hit All Values?Understanding the Existence and Uniqueness of the GCDGCD and LCM with logical symbolsThe greatest common divisor of two positive integers less than 100 is equal to 3. Their least common multiple is twelve times one of the integers.Suppose that for all integers $x$, $x|a$ and $x|b$ if and only if $x|c$. Then $c = gcd(a,b)$Which is the gcd of 2 numbers which are multiplied and the result is 600000?

            Ingelân Ynhâld Etymology | Geografy | Skiednis | Polityk en bestjoer | Ekonomy | Demografy | Kultuer | Klimaat | Sjoch ek | Keppelings om utens | Boarnen, noaten en referinsjes Navigaasjemenuwww.gov.ukOffisjele webside fan it regear fan it Feriene KeninkrykOffisjele webside fan it Britske FerkearsburoNederlânsktalige ynformaasje fan it Britske FerkearsburoOffisjele webside fan English Heritage, de organisaasje dy't him ynset foar it behâld fan it Ingelske kultuergoedYnwennertallen fan alle Britske stêden út 'e folkstelling fan 2011Notes en References, op dizze sideEngland

            Հադիս Բովանդակություն Անվանում և նշանակություն | Դասակարգում | Աղբյուրներ | Նավարկման ցանկ